« first day (1693 days earlier)      last day (2641 days later) » 

5:50 AM
@MartinSleziak I tried to remove once again.
 
 
4 hours later…
10:03 AM
1
Q: Explicit example and the continuum hypothesis

E. JosephI know that the continuum hypothesis is not decidable, i.e. we can not prove it, nor disprove it. The question is, is it theoretically possible to find an explicit set $E\subset \mathbb R$ such that we can not prove neither $\vert \mathbb N\vert =\vert E\vert$, nor $\vert \mathbb R\vert =\vert E...

 
 
3 hours later…
1:20 PM
@MartinSleziak I have added suggestion to remove the tag on meta. It is a bit unfortunate that the tag-creator cannot currently respond. But still, they might make the case for the tag later, if they think it is useful.
0
A: Tag management 2017

Martin SleziakI suggest to remove theory-of-equations tag. This tag was created in December. The tag creator also provided the tag-excerpt and the tag-wiki which more-or-less follow the Wikipedia article Theory of equations. Looking at the tag-info, it seems that most of the problems described there are cover...

 
1:38 PM
0
A: Tag management 2017

Martin SleziakThe tag a.m.-g.m.-inequality has been created recently Is this really going to be useful? It seems a bit too specific to me. In any case, it is probably worth discussing this on meta before the tag grows too large. I am not denying that the inequality is useful and well-known. However, it seem...

There are currently two questions tagged .
0
Q: Prove that $x+\frac{1}{x}\geq2$

Sathasivam KTo prove $x+\frac{1}{x}\geq2$ where $x$ is a positive real number. This is what i try: $$\text{We need to prove } \hspace{1cm} x+\frac{1}{x}-2\geq 0$$ now,$$\frac{x^2-2x+1}{x}=(x-2)+\frac{1}{x}$$ its enough to show that $$\frac{1}{x}\geq(x-2)\hspace{0.5cm} \text{ when } \hspace{0.2cm}0<x\leq 2$$ ...

1
Q: $a,b,c,d>0$ and $abcd=1$ prove $\sum \frac {1}{(1+a)(1+a^2)} \ge 1$

Iliya ShadfarIf $a,b,c,d>0$ and $abcd=1$ prove: $$\sum \frac{1}{(1+a)(1+a^2)}\ge 1$$ Here's my solution: I try to prove it by reverse: $$3\ge\sum\frac{a^3+a^2+a}{(a+1)(a^2+1)}$$Then by AM-GM: $$3\ge\sum\frac{a(1+a+a^2)}{2a(a+1)}$$$$$6\ge\sum\frac{1+a+a^2}{a+1}$$ Then We need to prove: $$2\ge\sum\frac{a^2}{a...

@HarshKumar I just wanted to let you know that I have made a post on meta about the tag (a.m.-g.m.-inequality) which you have recently created. — Martin Sleziak 7 secs ago
 

« first day (1693 days earlier)      last day (2641 days later) »